0 Daumen
168 Aufrufe


(a) \( n^{2} \leq 2^{n} \) für alle \( n \in \mathbb{N} \) mit \( n \geq 4 \),

(b) \( \sum \limits_{k=1}^{n} \frac{1}{k^{2}} \leq 2-\frac{1}{n} \) für alle \( n \in \mathbb{N} \).

Avatar von

2 Antworten

0 Daumen
 
Beste Antwort

Aloha :)

zu a)\(\quad n^2\le2^n\quad\text{für }n\ge4\)

Verankerung bei \(n=4\):\(\quad n^2=4^2=16=2^4\le2^n\quad\checkmark\)

Induktionsschritt von \(n\) auf \(n+1\), wobei \(n\ge4\):

Für \(n\ge3\) gilt:\(\quad (n-1)^2>2\implies n^2-2n+1>2\implies n^2>2n+1\)

Daher gilt:\(\quad(n+1)^2=n^2+2n+1<n^2+n^2=2\cdot n^2\stackrel{(\text{Ind.Vor.})}{\le}2\cdot2^n=2^{n+1}\;\checkmark\)


zu b)\(\quad\sum\limits_{k=1}^n\frac{1}{k^2}\le2-\frac1n\quad\text{für }n\in\mathbb N\)

Verankerung bei \(n=1\):\(\quad\sum_{k=1}^n\frac{1}{k^2}=\frac{1}{1^2}=1=2-\frac11=2-\frac1n\quad\checkmark\)

Induktionsschritt von \(n\) auf \(n+1\):

$$\sum\limits_{k=1}^{n+1}\frac{1}{k^2}=\frac{1}{(n+1)^2}+\sum\limits_{k=1}^n\frac{1}{k^2}<\frac{1}{n\cdot(n+1)}+\sum\limits_{k=1}^n\frac{1}{k^2}=\left(\frac1n-\frac{1}{n+1}\right)+\sum\limits_{k=1}^n\frac{1}{k^2}$$Jetzt verwenden wir die Induktionsvoraussetzung:$$\phantom{\sum\limits_{k=1}^{n+1}\frac{1}{k^2}}\le\left(\frac1n-\frac{1}{n+1}\right)+2-\frac1n=2-\frac{1}{n+1}\quad\checkmark$$

Avatar von 148 k 🚀
0 Daumen

i.A.

4^2 <= 2^4
16 <= 16 → stimmt

i.S.

(n + 1)^2 <= 2^(n + 1)
n^2 + 2n + 1 <= 2 * 2^n
n^2 + 2n + 1 <= 2^n + 2^n

n^2 <= 2^n → Folgt aus Induktionsvoraussetzung
2n + 1 <= n^2 <= 2^n --> Wäre für n >= 4 auch wahr

Avatar von 479 k 🚀

Ein anderes Problem?

Stell deine Frage

Willkommen bei der Mathelounge! Stell deine Frage einfach und kostenlos

x
Made by a lovely community